§2 平均値の定理

「§2 平均値の定理」の編集履歴(バックアップ)一覧はこちら

§2 平均値の定理」(2013/09/01 (日) 00:30:13) の最新版変更点

追加された行は緑色になります。

削除された行は赤色になります。

>編集中... &bold(){(極大、極小)} f(x) が x で&bold(){極大}とは、x を含む'''開'''区間 U が(小さくても良いから)存在して、 任意の y∈U に対して f(y)≦f(x) となることと定める。 f(x) が x で&bold(){極小}とは、(中略) f(y)≧f(x) となることと定める。 さらに、x≠y ならば f(x)<f(y) となるとき&bold(){狭義の極大}という。 狭義の極小についても同様。 #blockquote(){{{命題 2.3 (極値の必要条件) f(x) は微分可能とする。 f(x) が c で極大(小)となれば f'(c)=0}}} '''Proof.''' まず、$$ f'(c)=\lim_{h\to+0}\frac{f(c+h)-f(c)}{h} $$ 分母は負、分子は正なので、極限の f'(c) は0以下 また、$$ f'(c)=\lim_{h\to-0}\frac{f(c+h)-f(c)}{h} $$ 分母は負、分子は負なので、極限の f'(c) は0以上 したがって f'(c) は0以上0以下、すなわち0。 ∥ ---- これから3つの定理を示すが、これらは次のような関係を持っている。 >&bold(){定理 2.4 (ロルの定理)} >↓ 応用 >&bold(){定理 2.5 (平均値の定理)} >↓ 応用 >&bold(){定理 2.6 (コーシーの平均値の定理)} 逆に見れば、 >&bold(){定理 2.4 (ロルの定理)} >↑ f(a)=f(b) の場合 >&bold(){定理 2.5 (平均値の定理)} >↑ g(x)=x の場合 >&bold(){定理 2.6 (コーシーの平均値の定理)} という関係でもある。 #blockquote(){{{定理 2.4 (ロルの定理) f(x) が[a, b]で連続、(a, b)で微分可能とし、 f(a)=f(b) であるとする。 このとき、ある点 c∈(a, b) が存在して、f'(c)=0となる。}}} '''Proof.''' f(x) は連続なので、定理 1.24 (夏学期の大定理) より、[a, b]の点で最大値、最小値をとる。 f(x) が定数関数ならば f'(x)=0 はいたるところで成り立つ。 f(x) が定数関数でないときを考えよう。 最大値と最小値、どちらも(a, b)に存在しないと仮定すると、 最大値と最小値はどちらも x=a, b のどちらかに存在することになるが、 f(a)=f(b) より、最大値=最小値=f(a)。 つまり f(x) は定数関数となり矛盾する。 したがって、最大値と最小値の少なくとも一方は(a, b)に存在する。 それを c と書こう。 c は[a, b]の内点(端ではない)なので、最大なら極大、最小なら極小。 (命題 2.3 より、)どちらにしても f'(c)=0 が成り立つ。 ∥ #blockquote(){{{定理 2.5 (平均値の定理) f(x) が[a, b]で連続、(a, b)で微分可能とすると、 $$\frac{f(b)-f(a)}{b-a}=f'(c)$$ となる c∈(a, b) が存在する。}}} '''Proof.''' $$\phi(x)=f(x)-\frac{f(b)-f(a)}{b-a}x$$とおく。 >つまり、φ(x) は2点(a, f(a)), (b, f(b))を通る直線と、f(x)の差である。 このとき、 $$\begin{align}\phi(a)&=f(a)-\frac{f(b)-f(a)}{b-a}a=\frac{bf(a)-af(a)-af(b)+af(a)}{b-a}=\frac{bf(a)-af(b)}{b-a} \\ \phi(b)&=f(b)-\frac{f(b)-f(a)}{b-a}b=\frac{bf(b)-af(b)-bf(b)+bf(a)}{b-a}=\frac{bf(a)-af(b)}{b-a}\end{align}$$ なので、φ(a)=φ(b) よってロルの定理より、ある c∈(a, b) が存在して、φ'(c)=0 ところで φ'(x)を計算すると$$\phi'(x)=f'(x)-\frac{f(b)-f(a)}{b-a}$$なので、 φ'(c)=0 より$$f'(c)-\frac{f(b)-f(a)}{b-a}=0$$ ∥ #blockquote(){{{定理 2.6 (コーシーの平均値の定理) f, g が[a, b]で連続、(a, b)で微分可能とする。 さらに、g(a)≠g(b) で、f'(x) と g'(x) は同時に零にならないものとする。 このとき、 $$ \frac{f(b)-f(a)}{g(b)-g(a)}=\frac{f'(c)}{g'(c)} $$ となる c∈(a, b) が存在する。}}} '''Proof.''' $$ \phi(x)=(g(b)-g(a))f(x)-(f(b)-f(a))g(x) $$ とおくと、 $$ \phi(b)-\phi(a)=(g(b)-g(a))(f(b)-f(a))-(f(b)-f(a))(g(b)-g(a))=0 $$ よって 定理 2.5 (平均値の定理) が使えて、 φ'(c)=0 なる c∈(a, b) が存在する。 左辺を計算すると、 $$ \underbrace{(g(b)-g(a))}_{\ne 0}f'(c) = (f(b)-f(a))g'(c) $$ g'(c)=0 だと f'(c)=0 となり題意に反する。 よって g'(c)≠0 で、$$ \frac{f(b)-f(a)}{g(b)-g(a)}=\frac{f'(c)}{g'(c)} $$ ∥ ---- お待たせしました! 定理 2.6 (コーシーの平均値の定理) が得られたので次の定理が証明できます! #blockquote(){{{定理 2.7 (テイラーの定理) この定理に限り、[a, x], (a, x)は a>x のとき[x, a], (x, a)のこととする。 f(x) が[a, x]で連続、(a, x)で n 回微分可能とする。 $$ f(x)=f(a)+\frac{f'(a)}{1!}(x-a)+\frac{f' '(a)}{2!}(x-a)^2+\frac{f'''(a)}{3!}(x-a)^3+\cdots+\frac{f^{(n-1)}(a)}{(n-1)!}(x-a)^{n-1} \ + \ R_n $$ // f' 'にある空白は、太字にするコマンドに指定されている''を回避するためです。 $$ R_n=\frac{f^{(n)}(\xi)}{n!}(x-a)^n $$ をみたす ξ が(a, x)に存在する。 この$$R_n$$を剰余項、(またはラグランジュの剰余項)と呼ぶ。}}} '''Proof.''' φ(x)=R&sub(){n}, g(x)=(x-a)&sup(){n} とおくことで、 R&sub(){n} の係数の部分 φ(x)/g(x) について調べたい。 まず、φ(x), g(x) の導関数を計算しておく。 (横長ですいません。1本目の式の最後の項は(n-1)次です) #math(128){{{\renewcommand{\arraystretch}{2.0}\begin{array}{llr} \phi(x)&=f(x)&-f(a)-\frac{f'(a)}{1!}(x-a)-\frac{f' '(a)}{2!}(x-a)^2-\frac{f'''(a)}{3!}(x-a)^3-\cdots-\frac{f^{(n-1)}(a)}{(n-1)!}(x-a)^{n-1} \\ \phi'(x)&=f'(x)&-f'(a)\quad\quad\quad-\frac{f' '(a)}{1!}(x-a)-\frac{f'''(a)}{2!}(x-a)^2-\cdots-\frac{f^{(n-2)}(a)}{(n-2)!}(x-a)^{n-2} \\ \phi' '(x)&=f' '(x)&-f' '(a)\quad\quad\quad-\frac{f'''(a)}{1!}(x-a)\ -\cdots-\frac{f^{(n-3)}(a)}{(n-3)!}(x-a)^{n-3} \\ & \vdots & \vdots \quad\quad\quad\quad\quad\quad \\ \end{array} }}} $$\begin{align} & \phi^{(n-1)}(x)=f^{(n-1)}(x) -f^{(n-1)}(a) \\ & \phi^{(n)}(x)=f^{(n)}(x) \end{align} $$ $$\begin{array}{rcl} g(x) &=& (x-a)^n \\ g'(x) &=& n (x-a)^{n-1} \\ g''(x) &=& n(n-1) (x-a)^{n-2} \\ &\vdots& \\ g^{(n-1)}(x) &=& n(n-1) \cdots 2(x-a) \\ g^{(n)}(x) &=& n! \end{array}$$ 必要なものを計算してまとめると、 $$(*)\cdots\begin{cases} \phi(a)=\phi'(a)=\phi''(a)=\cdots=\phi^{(n-1)}(a)=0 \\ \phi^{(n)}(x)=f^{(n)}(x) \\ g(a)=g'(a)=g''(a)=\cdots=g^{(n-1)}(a)=0 \\ g^{(n)}(x)=n! \end{cases}$$ さて、ここから 定理 2.6 (コーシーの平均値の定理) を使う。 使えるかどうかのチェックは後からやります。 $$ \frac{\phi(x)}{g(x)} \overset{(*)}{=} \frac{\phi(x)-\phi(a)}{g(x)-g(a)} \overset{2.6}{=} \frac{\phi'(c_1)}{g'(c_1)} \overset{(*)}{=} \frac{\phi'(c_1)-\phi'(a)}{g'(c_1)-g'(a)} \overset{2.6}{=}\frac{\phi' '(c_2)}{g' '(c_2)} \overset{(*)}{=} \cdots $$   $$ \overset{2.6}{=} \frac{\phi^{(n-1)}(c_{n-1})}{g^{(n-1)}(c_{n-1})} \overset{(*)}{=} \frac{\phi^{(n-1)}(c_{n-1})-\phi^{(n-1)}(a)}{g^{(n-1)}(c_{n-1})-g^{(n-1)}(a)} \overset{2.6}{=} \frac{\phi^{(n)}(c_n)}{g^{(n)}(c_n)} \overset{(*)}{=} \frac{f^{(n)}(c_n)}{n!} $$ 定理 2.6 (コーシーの平均値の定理) をどう使ったかをチェックします。 1回目、「φ, g は[x, a]で連続、(x, a)で微分可能」である必要がありますが、 φ は f-(n-1次式) だからOK。g は明らかにOK。 さらに「g(x)≠g(a) で、φ' と g' は同時に零にならない」ですが、 g(x)≠g(a)はどうみてもOKです。 (*)より φ'(a)=g'(a)=0 ...おっと!? ……大丈夫です。コーシーの平均値の定理では φ' や g' は(x, a)の範囲でしか考えません。 この範囲では g' は零にはなりません。 φ'(a) や g'(a) は範囲外です。やったね! ゆえに 定理 2.6 は使えて、等号が成り立つ c&sub(){1}∈(x, a) が存在する。 2回目、区間は(c&sub(){1}, a)になりました。 g'(c&sub(){1})≠g'(a) (=0) であり、g'' は零とならない。OKです。 ゆえに等号が成り立つ c&sub(){2}∈(c&sub(){1}, a) が存在する。 (n-1)回目までは同様にOKです。 n 回目、g&sup(){(n)} は零とならないOKです。 ゆえに等号が成り立つ c&sub(){n}∈(c&sub(){n-1}, a) が存在する。 ということで、 $$ \frac{\phi(x)}{g(x)} = \frac{f^{(n)}(c_n)}{n!} $$ となる c&sub(){n}∈(c&sub(){n-1}, a)が存在する。 c&sub(){1}∈(x, a) c&sub(){2}∈(c&sub(){1}, a) c&sub(){3}∈(c&sub(){2}, a) ... c&sub(){n}∈(c&sub(){n-1}, a) なので、c&sub(){n}∈(x, a)が成り立つ。 この c&sub(){n} こそが ξ である! $$ \frac{R_n}{g(x)}=\frac{\phi(x)}{g(x)}=\frac{f^{(n)}(c_n)}{n!}=\frac{f^{(n)}(\xi)}{n!}$$より、 $$ R_n = \frac{f^{(n)}(\xi)}{n!}(x-a)^n $$                           ∥ お疲れ様でした、それではテイラー展開に進みましょう。 ---- ...... #co(){{{{{{ f(x, y) は(全)微分可能とする。 t の一変数関数 φ(t) を、 $$\phi (t)=f(a+\alpha t,b+\beta t)$$ と定める。 // φ(t) = f(a+αt, b+βt) と定める。 >つまり、φ(t) は xy 平面上の点(a, b)から速度(α, β)で t 進んだところの f(x, y) の値。 >φ(t) は f(x, y) の断面となっている。 φ(t) を t で微分してみる。  $$\begin{align} \phi '(t) & =\lim_{h \to 0} \frac{1}{h}\{ \phi (t+h)-\phi (t) \} \\ & =\lim_{h \to 0} \frac{1}{h}\{ f(a+\alpha t+\alpha h, b+\beta t+\beta h)-f(a+\alpha t, b+\beta t) \} \end{align} $$ ここで、(a+αt+αh, b+βt+βh) は (a+αt, b+βt) から (αh, βh) だけずれた点だと考えると、 f は微分可能なので、  $$\begin{align} f(a+\alpha t+\alpha h, b+\beta t+\beta h) &= f(a+\alpha t, b+\beta t) \\ & + \frac{\partial f(a+\alpha t, b+\beta t)}{\partial x}\cdot \alpha h + \frac{\partial f(a+\alpha t, b+\beta t)}{\partial y}\cdot \beta h + \epsilon (\alpha h, \beta h)\end{align}$$ よって  $$\begin{align} \phi '(t) & = \lim_{h \to 0} \frac{1}{h} \left\{ \frac{\partial f(a+\alpha t, b+\beta t)}{\partial x}\cdot \alpha h + \frac{\partial f(a+\alpha t, b+\beta t)}{\partial y}\cdot \beta h + \epsilon (\alpha h, \beta h) \right\} \\ & = \frac{\partial f(a+\alpha t, b+\beta t)}{\partial x}\cdot \alpha + \frac{\partial f(a+\alpha t, b+\beta t)}{\partial y}\cdot \beta + \lim_{h \to 0} \frac{ \epsilon (\alpha h, \beta h) }{h} \end{align} $$ ここで、第3項は0である。   >なぜならば、    $$\lim_{h \to 0} \frac{ \epsilon (\alpha h, \beta h) }{h}=\lim_{h \to 0} \underbrace{ \frac{ \epsilon (\alpha h, \beta h) }{\sqrt{\alpha^2h^2+\beta^2h^2}} }_{\to 0} \cdot \underbrace{ \frac{\sqrt{\alpha^2h^2+\beta^2h^2}}{h} }_{=\sqrt{\alpha^2+\beta^2}}$$   「→0」は、εの定義による。 まとめると、 #blockquote(){{{(方向微分) $$\phi'(t)=\left(\alpha\tfrac{\partial}{\partial x}+\beta\tfrac{\partial}{\partial y}\right)f(a+\alpha t, b+\beta t)$$ }}} これは、関数 f の 点(x, y) の場所を ベクトル(α, β) の方向になぞったときの傾きである。 φ(t) をもっと微分してみる。 #blockquote(){{{命題 2.17 (高階の方向微分の二項展開) f(x, y) は十分滑らか、すなわち高階の偏導関数$$(\tfrac{\partial}{\partial x})^k(\tfrac{\partial}{\partial y})^{m-k}f(x, y)$$が存在して連続であるとする。(m≫1, k=0, 1, ... ,m) (つまり、偏導関数 ∂/∂x と ∂/∂y の順序が交換できる。) このとき、$$ \left(\frac{d}{dt}\right)^m\phi(t) = \phi^{(m)}(t) = \left(\alpha\frac{\partial}{\partial x}+\beta\frac{\partial}{\partial y}\right)^m f(a+\alpha t, b+\beta t)$$が成り立つ。 ここで、$$(\alpha\tfrac{\partial}{\partial x}+\beta\tfrac{\partial}{\partial y})^mf(x, y)$$は $$\sum^m_{k=0}\alpha^k\beta^{m-k}\binom{m}{k}\left(\frac{\partial}{\partial x}\right)^k\left(\frac{\partial}{\partial y}\right)^{m-k}\!\!f(x, y) $$の略記である。 $$\tbinom{m}{k}$$は二項係数&sub(){m}C&sub(){k}のことである。}}} '''Proof.''' >f(x, y) が十分滑らかなとき $$\tfrac{\partial f}{\partial x}(x, y)$$ もまた十分滑らかな2変数関数であると分かっていれば難なく示せる。 m=k のとき成り立つと仮定する。 $$g(x, y)=(\alpha\tfrac{\partial}{\partial x}+\beta\tfrac{\partial}{\partial y})f(x, y)$$ とおくと、 gは十分滑らかで、$$\tfrac{d}{dt}\phi(t)=g(a+\alpha t, b+\beta t)$$ が成り立つ。 さらに、$$\psi(t)=g(a+\alpha t, b+\beta t)$$ とおくと、 仮定より、$$\tfrac{d^{k}}{dt^{k}}\psi(t)=(\alpha\tfrac{\partial}{\partial x}+\beta\tfrac{\partial}{\partial y})^k g(a+\alpha t, b+\beta t)$$ したがって、 $$\tfrac{d^{k+1}}{dt^{k+1}}(t) = \tfrac{d^{k}}{dt^{k}}(\tfrac{d}{dt}\phi(t)) = \tfrac{d^{k}}{dt^{k}}g(a+\alpha t, b+\beta t) = \tfrac{d^{k}}{dt^{k}}\psi(t)$$ 仮定より、  $$\begin{align}&=(\alpha\tfrac{\partial}{\partial x}+\beta\tfrac{\partial}{\partial y})^k g(a+\alpha t, b+\beta t) \\ &=(\alpha\tfrac{\partial}{\partial x}+\beta\tfrac{\partial}{\partial y})^k (\alpha\tfrac{\partial}{\partial x}+\beta\tfrac{\partial}{\partial y}) f(a+\alpha t, b+\beta t) \\ &= (\alpha\tfrac{\partial}{\partial x}+\beta\tfrac{\partial}{\partial y})^{k+1} f(a+\alpha t, b+\beta t)\end{align}$$ となり、 m=k+1 でも成り立つ。 帰納法で示された。 ∥ φ(t)を0を中心に展開するテイラーの定理は以下のようである。 ある実数 0<θ<1 が存在して、 $$\phi(t)=\phi(0)+\phi'(0)t+\frac{\phi''(0)}{2!}t^2+\cdots+\frac{\phi^{(n-1)}(0)}{(n-1)!}t^{n-1}+\frac{\phi^{n}(\theta t)}{n!}t^n$$ ここで、t=1とし、命題 2.17 を適用すると次の定理が得られる。 #blockquote(){{{命題 2.18 (2変数のテーラーの公式) $$ f(a+\alpha, b+\beta)=f(a, b)+(\alpha\tfrac{\partial}{\partial x}+\beta\tfrac{\partial}{\partial y})f(a, b)+\frac{1}{2!}(\alpha\tfrac{\partial}{\partial x}+\beta\tfrac{\partial}{\partial y})^2f(a, b)+\cdots+\frac{1}{(n-1)!}(\alpha\tfrac{\partial}{\partial x}+\beta\tfrac{\partial}{\partial y})^{n-1}f(a, b)\ \ +\frac{1}{n!}(\alpha\tfrac{\partial}{\partial x}+\beta\tfrac{\partial}{\partial y})^nf(a+\theta\alpha, b+\theta\beta) $$ を満たす実数 0<θ<1 が存在する。}}} ---- 極大、極小、峠ではどれも f&sub(){x}=f&sub(){y}=0 となる。 ではどのように区別できるだろうか。 点(a, b) で f&sub(){x}=f&sub(){y}=0 となるとき、点(a, b) を中心とした f(x, y) のテイラー展開を考えると、 $$ f(a+\alpha, b+\beta)=f(a, b)+(0+0)+\frac{1}{2}(\alpha^2 f_{xx}+2\alpha\beta f_{xy}+\beta^2 f_{yy})(a, b)+\cdots $$ (α, βの高次項)  $$ \approx f(a, b)+\frac{1}{2}\{\alpha^2 f_{xx}(a, b)+2\alpha\beta f_{xy}(a, b)+\beta^2 f_{yy}(a, b)\}$$ つまり、αやβの係数である f&sub(){xx}(a, b), f&sub(){xy}(a, b), f&sub(){yy}(a, b) が 点(a, b) 付近での f の局所的な構造を決定しているのである。 >f&sub(){xx}(a, b), f&sub(){xy}(a, b), f&sub(){yy}(a, b) の値を見ただけで極大、極小、峠を区別できたらうれしい……そこで、次は2次形式を学びます。 ---- *次:§7 2次形式 }}}}}}
&bold(){(極大、極小)} f(x) が x で&bold(){極大}とは、x を含む'''開'''区間 U が(小さくても良いから)存在して、 任意の y∈U に対して f(y)≦f(x) となることと定める。 f(x) が x で&bold(){極小}とは、(中略) f(y)≧f(x) となることと定める。 さらに、x≠y ならば f(x)<f(y) となるとき&bold(){狭義の極大}という。 狭義の極小についても同様。 #blockquote(){{{命題 2.3 (極値の必要条件) f(x) は微分可能とする。 f(x) が c で極大(小)となれば f'(c)=0}}} '''Proof.''' まず、$$ f'(c)=\lim_{h\to+0}\frac{f(c+h)-f(c)}{h} $$ 分母は負、分子は正なので、極限の f'(c) は0以下 また、$$ f'(c)=\lim_{h\to-0}\frac{f(c+h)-f(c)}{h} $$ 分母は負、分子は負なので、極限の f'(c) は0以上 したがって f'(c) は0以上0以下、すなわち0。 ∥ ---- これから3つの定理を示すが、これらは次のような関係を持っている。 >&bold(){定理 2.4 (ロルの定理)} >↓ 応用 >&bold(){定理 2.5 (平均値の定理)} >↓ 応用 >&bold(){定理 2.6 (コーシーの平均値の定理)} 逆に見れば、 >&bold(){定理 2.4 (ロルの定理)} >↑ f(a)=f(b) の場合 >&bold(){定理 2.5 (平均値の定理)} >↑ g(x)=x の場合 >&bold(){定理 2.6 (コーシーの平均値の定理)} という関係でもある。 #blockquote(){{{定理 2.4 (ロルの定理) f(x) が[a, b]で連続、(a, b)で微分可能とし、 f(a)=f(b) であるとする。 このとき、ある点 c∈(a, b) が存在して、f'(c)=0となる。}}} '''Proof.''' f(x) は連続なので、定理 1.24 (夏学期の大定理) より、[a, b]の点で最大値、最小値をとる。 f(x) が定数関数ならば f'(x)=0 はいたるところで成り立つ。 f(x) が定数関数でないときを考えよう。 最大値と最小値、どちらも(a, b)に存在しないと仮定すると、 最大値と最小値はどちらも x=a, b のどちらかに存在することになるが、 f(a)=f(b) より、最大値=最小値=f(a)。 つまり f(x) は定数関数となり矛盾する。 したがって、最大値と最小値の少なくとも一方は(a, b)に存在する。 それを c と書こう。 c は[a, b]の内点(端ではない)なので、最大なら極大、最小なら極小。 (命題 2.3 より、)どちらにしても f'(c)=0 が成り立つ。 ∥ #blockquote(){{{定理 2.5 (平均値の定理) f(x) が[a, b]で連続、(a, b)で微分可能とすると、 $$\frac{f(b)-f(a)}{b-a}=f'(c)$$ となる c∈(a, b) が存在する。}}} '''Proof.''' $$\phi(x)=f(x)-\frac{f(b)-f(a)}{b-a}x$$とおく。 >つまり、φ(x) は2点(a, f(a)), (b, f(b))を通る直線と、f(x)の差である。 このとき、 $$\begin{align}\phi(a)&=f(a)-\frac{f(b)-f(a)}{b-a}a=\frac{bf(a)-af(a)-af(b)+af(a)}{b-a}=\frac{bf(a)-af(b)}{b-a} \\ \phi(b)&=f(b)-\frac{f(b)-f(a)}{b-a}b=\frac{bf(b)-af(b)-bf(b)+bf(a)}{b-a}=\frac{bf(a)-af(b)}{b-a}\end{align}$$ なので、φ(a)=φ(b) よってロルの定理より、ある c∈(a, b) が存在して、φ'(c)=0 ところで φ'(x)を計算すると$$\phi'(x)=f'(x)-\frac{f(b)-f(a)}{b-a}$$なので、 φ'(c)=0 より$$f'(c)-\frac{f(b)-f(a)}{b-a}=0$$ ∥ #blockquote(){{{定理 2.6 (コーシーの平均値の定理) f, g が[a, b]で連続、(a, b)で微分可能とする。 さらに、g(a)≠g(b) で、f'(x) と g'(x) は同時に零にならないものとする。 このとき、 $$ \frac{f(b)-f(a)}{g(b)-g(a)}=\frac{f'(c)}{g'(c)} $$ となる c∈(a, b) が存在する。}}} '''Proof.''' $$ \phi(x)=(g(b)-g(a))f(x)-(f(b)-f(a))g(x) $$ とおくと、 $$ \phi(b)-\phi(a)=(g(b)-g(a))(f(b)-f(a))-(f(b)-f(a))(g(b)-g(a))=0 $$ よって 定理 2.5 (平均値の定理) が使えて、 φ'(c)=0 なる c∈(a, b) が存在する。 左辺を計算すると、 $$ \underbrace{(g(b)-g(a))}_{\ne 0}f'(c) = (f(b)-f(a))g'(c) $$ g'(c)=0 だと f'(c)=0 となり題意に反する。 よって g'(c)≠0 で、$$ \frac{f(b)-f(a)}{g(b)-g(a)}=\frac{f'(c)}{g'(c)} $$ ∥ ---- お待たせしました! 定理 2.6 (コーシーの平均値の定理) が得られたので次の定理が証明できます! #blockquote(){{{定理 2.7 (テイラーの定理) この定理に限り、[a, x], (a, x)は a>x のとき[x, a], (x, a)のこととする。 f(x) が[a, x]で連続、(a, x)で n 回微分可能とする。 $$ f(x)=f(a)+\frac{f'(a)}{1!}(x-a)+\frac{f' '(a)}{2!}(x-a)^2+\frac{f'''(a)}{3!}(x-a)^3+\cdots+\frac{f^{(n-1)}(a)}{(n-1)!}(x-a)^{n-1} \ + \ R_n $$ // f' 'にある空白は、太字にするコマンドに指定されている''を回避するためです。 $$ R_n=\frac{f^{(n)}(\xi)}{n!}(x-a)^n $$ をみたす ξ が(a, x)に存在する。 この$$R_n$$を剰余項、(またはラグランジュの剰余項)と呼ぶ。}}} '''Proof.''' φ(x)=R&sub(){n}, g(x)=(x-a)&sup(){n} とおくことで、 R&sub(){n} の係数の部分 φ(x)/g(x) について調べたい。 まず、φ(x), g(x) の導関数を計算しておく。 (横長ですいません。1本目の式の最後の項は(n-1)次です) #math(128){{{\renewcommand{\arraystretch}{2.0}\begin{array}{llr} \phi(x)&=f(x)&-f(a)-\frac{f'(a)}{1!}(x-a)-\frac{f' '(a)}{2!}(x-a)^2-\frac{f'''(a)}{3!}(x-a)^3-\cdots-\frac{f^{(n-1)}(a)}{(n-1)!}(x-a)^{n-1} \\ \phi'(x)&=f'(x)&-f'(a)\quad\quad\quad-\frac{f' '(a)}{1!}(x-a)-\frac{f'''(a)}{2!}(x-a)^2-\cdots-\frac{f^{(n-2)}(a)}{(n-2)!}(x-a)^{n-2} \\ \phi' '(x)&=f' '(x)&-f' '(a)\quad\quad\quad-\frac{f'''(a)}{1!}(x-a)\ -\cdots-\frac{f^{(n-3)}(a)}{(n-3)!}(x-a)^{n-3} \\ & \vdots & \vdots \quad\quad\quad\quad\quad\quad \\ \end{array} }}} $$\begin{align} & \phi^{(n-1)}(x)=f^{(n-1)}(x) -f^{(n-1)}(a) \\ & \phi^{(n)}(x)=f^{(n)}(x) \end{align} $$ $$\begin{array}{rcl} g(x) &=& (x-a)^n \\ g'(x) &=& n (x-a)^{n-1} \\ g''(x) &=& n(n-1) (x-a)^{n-2} \\ &\vdots& \\ g^{(n-1)}(x) &=& n(n-1) \cdots 2(x-a) \\ g^{(n)}(x) &=& n! \end{array}$$ 必要なものを計算してまとめると、 $$(*)\cdots\begin{cases} \phi(a)=\phi'(a)=\phi''(a)=\cdots=\phi^{(n-1)}(a)=0 \\ \phi^{(n)}(x)=f^{(n)}(x) \\ g(a)=g'(a)=g''(a)=\cdots=g^{(n-1)}(a)=0 \\ g^{(n)}(x)=n! \end{cases}$$ さて、ここから 定理 2.6 (コーシーの平均値の定理) を使う。 使えるかどうかのチェックは後からやります。 $$ \frac{\phi(x)}{g(x)} \overset{(*)}{=} \frac{\phi(x)-\phi(a)}{g(x)-g(a)} \overset{2.6}{=} \frac{\phi'(c_1)}{g'(c_1)} \overset{(*)}{=} \frac{\phi'(c_1)-\phi'(a)}{g'(c_1)-g'(a)} \overset{2.6}{=}\frac{\phi' '(c_2)}{g' '(c_2)} \overset{(*)}{=} \cdots $$   $$ \overset{2.6}{=} \frac{\phi^{(n-1)}(c_{n-1})}{g^{(n-1)}(c_{n-1})} \overset{(*)}{=} \frac{\phi^{(n-1)}(c_{n-1})-\phi^{(n-1)}(a)}{g^{(n-1)}(c_{n-1})-g^{(n-1)}(a)} \overset{2.6}{=} \frac{\phi^{(n)}(c_n)}{g^{(n)}(c_n)} \overset{(*)}{=} \frac{f^{(n)}(c_n)}{n!} $$ 定理 2.6 (コーシーの平均値の定理) をどう使ったかをチェックします。 1回目、「φ, g は[x, a]で連続、(x, a)で微分可能」である必要がありますが、 φ は f-(n-1次式) だからOK。g は明らかにOK。 さらに「g(x)≠g(a) で、φ' と g' は同時に零にならない」ですが、 g(x)≠g(a)はどうみてもOKです。 (*)より φ'(a)=g'(a)=0 ...おっと!? ……大丈夫です。コーシーの平均値の定理では φ' や g' は(x, a)の範囲でしか考えません。 この範囲では g' は零にはなりません。 φ'(a) や g'(a) は範囲外です。やったね! ゆえに 定理 2.6 は使えて、等号が成り立つ c&sub(){1}∈(x, a) が存在する。 2回目、区間は(c&sub(){1}, a)になりました。 g'(c&sub(){1})≠g'(a) (=0) であり、g'' は零とならない。OKです。 ゆえに等号が成り立つ c&sub(){2}∈(c&sub(){1}, a) が存在する。 (n-1)回目までは同様にOKです。 n 回目、g&sup(){(n)} は零とならないOKです。 ゆえに等号が成り立つ c&sub(){n}∈(c&sub(){n-1}, a) が存在する。 ということで、 $$ \frac{\phi(x)}{g(x)} = \frac{f^{(n)}(c_n)}{n!} $$ となる c&sub(){n}∈(c&sub(){n-1}, a)が存在する。 c&sub(){1}∈(x, a) c&sub(){2}∈(c&sub(){1}, a) c&sub(){3}∈(c&sub(){2}, a) ... c&sub(){n}∈(c&sub(){n-1}, a) なので、c&sub(){n}∈(x, a)が成り立つ。 この c&sub(){n} こそが ξ である! $$ \frac{R_n}{g(x)}=\frac{\phi(x)}{g(x)}=\frac{f^{(n)}(c_n)}{n!}=\frac{f^{(n)}(\xi)}{n!}$$より、 $$ R_n = \frac{f^{(n)}(\xi)}{n!}(x-a)^n $$                           ∥ お疲れ様でした、それではテイラー展開に進みましょう。 ---- ここからは[[テーラー展開の例]]に進んでください。

表示オプション

横に並べて表示:
変化行の前後のみ表示: